\documentclass[a5paper,10pt]{article}
\usepackage{myXsim}
\usepackage{tasks}

% Title Page
\title{DM2 \hfill BERTAN Ufuk}
\tribe{TST}
\date{\hfillÀ render pour le Mercredi 24 février}

\xsimsetup{
    solution/print = false
}

\begin{document}
\maketitle

\begin{exercise}[subtitle={Loi binomiale}]
    Trois personnes s'apprêtent à passer le portique de sécurité. On suppose que pour chaque personne la probabilité que le portique sonne est égale à $0.04$.

    Soit $X$ la variable aléatoire donnant le nombre de personnes faisant sonner le portique, parmi les 3 personnes de ce groupe.
    \begin{enumerate}
        \item Tracer l'arbre représentant le situation.
        \item Justifier que $X$ suit une loi binomiale dont on précisera les paramètres.
        \item Quelle est la probabilité qu'une seule personne fasse sonner le portique?
        \item Calculer puis interpréter les probabilités suivantes
            \[
                P(X = 0) \qquad \qquad P(X \geq 2)
            \]
        \item Calculer l'espérance de $X$ et interpréter le résultat.
    \end{enumerate}
\end{exercise}

\begin{solution}
    \begin{enumerate}
        \item 
            \begin{tikzpicture}[sloped]
                \node {.} 
                child {node {$0$}
                    child {node {$0$} 
                        child {node {$0$} 
                            edge from parent
                            node[above] {0.96}
                        }
                        child {node {$1$}
                            edge from parent
                            node[above] {0.04}
                        } 
                        edge from parent
                        node[above] {0.96}
                    }
                    child[missing] {}
                    child {node {$1$}
                        child {node {$0$} 
                            edge from parent
                            node[above] {0.96}
                        }
                        child {node {$1$}
                            edge from parent
                            node[above] {0.04}
                        } 
                        edge from parent
                        node[above] {0.96}
                    } 
                    edge from parent
                    node[above] {0.96}
                }
                child[missing] {}
                child[missing] {}
                child[missing] {}
                child { node {$1$}
                    child {node {$0$}
                        child {node {$0$} 
                            edge from parent
                            node[above] {0.96}
                        }
                        child {node {$1$}
                            edge from parent
                            node[above] {0.04}
                        } 
                        edge from parent
                        node[above] {0.96}
                    }
                    child[missing] {}
                    child {node {$1$}
                        child {node {$0$} 
                            edge from parent
                            node[above] {0.96}
                        }
                        child {node {$1$}
                            edge from parent
                            node[above] {0.04}
                        } 
                        edge from parent
                        node[above] {0.96}
                    } 
                    edge from parent
                    node[above] {0.04}
                } ;
            \end{tikzpicture}
        \item Chaque personne a 2 possibilités (1: fait sonner ou 2: ne fait pas sonner) et l'on fait passer 3 personnes ce qui correspond à une répétition identique et aléatoire. On peut donc modéliser la situation par une loi binomiale.
            \[
                X \sim \mathcal{B}(3; 0.76)
            \]
        \item Probabilité qu'une seule personne fasse sonner le portique. On voit qu'il y a 3 branches qui correspondent à cette situation dont
            \[
                P(X = 1) = 3 \times 0.04^1 \times 0.96^2  \approx 0.111
            \]
        \item 
            \[
                P(X = 0) = 0.96^3  \approx 0.885
            \]
            \[
                P(X \geq 2) = P(X = 2) + P(X = 3) =  3 \times 0.04^2 \times 0.96^1 + 0.04^3  \approx 0.005
            \]

        \item Il faut d'abord tracer le tableau résumant la loi de probabilité:
            \begin{center}
            \begin{tabular}{|c|*{4}{c|}}
                \hline
                Valeur & 0 & 1 & 2 & 3 \\
                \hline
                Probabilité & $0.885$ & $0.111$ & $0.005$ &$0.0$ \\
                \hline
            \end{tabular}
            \end{center}
            On peut alors calculer l'espérance
            \[
                E[X] = 0 \times 0.885 + 1 \times 0.111 + 2 \times 0.005 + 3 \times 0.0 = 0.12
            \]
            On peut donc estimer qu'il y aura en moyenne $0.12$ personnes qui feront sonner le portique sur les 3 personnes.
    \end{enumerate}
\end{solution}

\begin{exercise}[subtitle={Équation puissance}]
    Résoudre les équations et inéquations suivantes
    \begin{multicols}{2}
        \begin{enumerate}
            \item $10^x = 14$
            \item $11^x = 35$
            \item $0.05^x \leq 24$
            \item $4 \times 0.92^x = 47$
        \end{enumerate}
    \end{multicols}
\end{exercise}

\begin{solution}
    Les solutions ci-dessous ne sont pas justifiée car l'ordinateur ne sait pas faire. Par contre, vous vous devez savoir justifier vos réponses!
    \begin{enumerate}
        \item $x = \log(14)$
        \item $x = \frac{\log(35)}{\log(11)}$
        \item Il faut faire attention quand on divise par un log car ce dernier peut être négatif ce qui est le cas ici. Il faut donc pense à changer le sens de l'inégalité.

            $x \geq \frac{\log(24)}{\log(0.05)}$

        \item Il faut penser à faire la division à par $4$ avant d'utiliser le log car sinon, on ne peut pas utiliser la formule $\log(a^n) = n\times \log(a)$.

            $x = \frac{\log(11.75)}{\log(0.92)}$
    \end{enumerate}
\end{solution}

\begin{exercise}[subtitle={Étude de fonctions}]
    Soit $f(x) = 4x^3 - 306x^2 + 4128x + 39$ une fonction définie sur $\R$.
    \begin{enumerate}
        \item Calculer $f'(x)$ la dérivée de $f(x)$.
        \item Calculer $f'(43)$ et $f'(8)$.
        \item En déduire une forme factorisée de $f'(x)$.
        \item Étudier le signe de $f'(x)$ et en déduire les variations de $f(x)$.
        \item Est-ce que la fonction $f(x)$ admet un maximum ou un minimum? Si oui, calculer sa valeur.
    \end{enumerate}
\end{exercise}

\begin{solution}
    \begin{enumerate}
        \item Dérivée de $f(x)$: $f'(x) = 12x^2 - 612x + 4128$
        \item 
            \begin{align*}
                f'(43) &= 12 \times 43^{2} - 612 \times 43 + 4128\\&= 12 \times 1849 - 26316 + 4128\\&= 22188 - 22188\\&= 0
            \end{align*}
            \begin{align*}
                f'(8) &= 12 \times 8^{2} - 612 \times 8 + 4128\\&= 12 \times 64 - 4896 + 4128\\&= 768 - 768\\&= 0
            \end{align*}
            Donc $x = 43$ et $x=8$ sont des racines de $f'(x) = 12x^2 - 612x + 4128$.
        \item On en déduit la forme factorisée suivante
            \[
                f'(x) = 12 (x - 43)(x-8)
            \]
        \item Pas de correction disponible
        \item À causes des branches extérieurs, la fonction $f(x)$ n'a pas de maximum ou de minimum.
    \end{enumerate}
\end{solution}



%\printsolutionstype{exercise}



\end{document}

%%% Local Variables: 
%%% mode: latex
%%% TeX-master: "master"
%%% End: